Math, asked by gopalnaveen27, 3 months ago

Find the four numbers forming a G.P if the second number exceeds
the first by 21 and the fourth number is 336 more than the third.

Answers

Answered by MaheswariS
3

\textbf{Given:}

\textsf{In a G.P, Second term exceeds the first term by 21 and}

\textsf{fourth number is 336 more than the third}

\textbf{To find:}

\textsf{The G.P}

\textbf{Solution:}

\textbf{Formula\;used:}

\mathsf{The\;n\,th\;term\;of\;G.P,\;\;a,ar,ar^2,.\,.\,.\;is}

\boxed{\mathsf{t_n=ar^{n-1}}}

\mathsf{Let\;the\;four\;numbers\;in\;G.P\;be\;a,\;ar,\;ar^2,\;ar^3}

\textsf{As per given data,}

\mathsf{t_2-t_1=21}

\implies\mathsf{ar-a=21}------(1)

\mathsf{Also,}

\mathsf{t_4-t_3=336}

\implies\mathsf{ar^3-ar^2=336}

\implies\mathsf{r^2(ar-a)=336}

\implies\mathsf{r^2(21)=336}

\implies\mathsf{r^2=\dfrac{336}{21}}

\implies\mathsf{r^2=16}

\implies\mathsf{r=4}

\mathsf{From\;(1)}

\mathsf{4a-a=21}

\mathsf{3a=21}

\mathsf{a=\dfrac{21}{3}}

\implies\boxed{\mathsf{a=7}}

\therefore\textsf{The required numbers are}

\mathsf{7,\;28,\;112,\;448}

Answered by amitnrw
7

Given : the four numbers forming a G.P

the second number exceeds

the first by 21 and the fourth number is 336 more than the third

To Find : four numbers forming a G.P

Solution:

four numbers forming a G.P

a , ar , ar² , ar³

second number exceeds the first by 21

=> ar - a  = 21

=> a(r - 1)  = 21

fourth number is 336 more than the third.

=> ar³ - ar² = 336

=> ar²(r - 1) = 336

=> ar²(r - 1) /  a(r - 1)  = 336/21

=>  r² = 16

=> r = ± 4

Case 1 :  r = 4

  a(r - 1)  = 21

=> a(4 - 1) = 21

=> a = 7

Numbers are   7  ,  28  , 112   , 448

Case 2 :  r = -4

  a(r - 1)  = 21

=> a(-4 - 1) = 21

=> a = -21/5

Numbers are   -21/5  ,  84/5    , -336/5   , 1344/5

four numbers are :

 7  ,  28  , 112   , 448    or

-21/5  ,  84/5    , -336/5   , 1344/5

Learn More:

Find three numbers in G.P. such that their sum is 21 and sum of ...

https://brainly.in/question/12377053

In a gp of real number it is given that t1 t2 t3 t4= 30 t1 squares t2 ...

https://brainly.in/question/12373150

Similar questions